LSAT and Law School Admissions Forum

Get expert LSAT preparation and law school admissions advice from PowerScore Test Preparation.

 Administrator
PowerScore Staff
  • PowerScore Staff
  • Posts: 8916
  • Joined: Feb 02, 2011
|
#38154
Complete Question Explanation
(The complete setup for this game can be found here: lsat/viewtopic.php?t=14979)

The correct answer choice is (A)

To attack this question, use proper List question technique: take one rule and apply it to all answer choices, then take another rule, and apply it to the remaining answer choices, and so on, until only one answer choice remains. However, do not just take the rules in the order given. Instead, choose the rules to apply in order of the ease of seeing them visually inside each answer choice. This approach has one caveat: if you were able to make a powerful inference whose violation is easy to detect, examine the application of the inference first! In this game, we were able to infer that O is fourth, which immediately eliminates answer choice (E). The rest of the rules should be applied in this order: third, fourth, first, and second.

Answer choice (A): This is the correct answer choice.

Answer choice (B): This answer choice violates the third rule and is therefore incorrect.

Answer choice (C): This answer choice violates the first rule and is therefore incorrect.

Answer choice (D): This answer choice violates the second rule and is therefore incorrect.

Answer choice (E): This answer choice violates the fourth rule (along with our our main inference), and is therefore incorrect.

As usual, each of the four incorrect answer choices violates a different rule.

Get the most out of your LSAT Prep Plus subscription.

Analyze and track your performance with our Testing and Analytics Package.